1answer.
Ask question
Login Signup
Ask question
All categories
  • English
  • Mathematics
  • Social Studies
  • Business
  • History
  • Health
  • Geography
  • Biology
  • Physics
  • Chemistry
  • Computers and Technology
  • Arts
  • World Languages
  • Spanish
  • French
  • German
  • Advanced Placement (AP)
  • SAT
  • Medicine
  • Law
  • Engineering
Romashka [77]
2 years ago
9

raul works at a movie theatre. the function f(x) represents the amount of money raul earns per ticket, where x is the number of

tickets he sells. the function g(x) represents the number of tickets raul sells per hour
Mathematics
1 answer:
leonid [27]2 years ago
3 0

Question:

Raul works at a movie theatre. The function f(x) represents the amount of money Raul earns per ticket, where x is the number of tickets he sells. The function g(x) represents the number of tickets Raul sells per hour, where x is the number of hours he works. Show all work to find f(g(x)), and explain what f(g(x)) represents.

f(x) = 2x2 + 16

g of x equals the square root of the quantity 5 x cubed

Answer:

f(g(x))=  

Step-by-step explanation:

Given:

f(x) = 2x^2 + 16

g(x) = \sqrt{(5x^3)}

f(x) represents the amount of money Raul earns per ticket

x is the number of tickets he sells

g(x) represents the number of tickets Raul sells per hour

To find:

f(g(x)) = ?

Solution:

f(g(x)) represent the amount Raul earns by selling tickets for x hours

We know,

f(x)  = 2x^2 + 16

f(g(x)) =2(\sqrt{(5x^3)} )^2 +16

The square root and power 2 cancel each other

f(g(x))  = 2(5x^3 ) +16

f(g(x)) = 10x^3 +16

So at x hours

Raul earns 10x^3 +16

For example lets assume Raul works for 5 hours, then the amount he earns will be

f(g(x)) = 10(5)^3 +16

f(g(x))  = 10(125) +16

f(g(x)) = 1250 +16

f(g(x)) =  1266

You  can also find the amount Raul earns by

g(x) = \sqrt{(5(5)^3)}

g(x) = \sqrt{625}

g(x) = 25

Substituting in f(x)

f(g(x)) = f(25)

f(25)  = 2(25)^2 + 16

f(25) = 2(625) +16

f(25) = 1250 + 16

f(25) = 1266

so Raul will be earning 1266 if he sells tickets for 5 hours

You might be interested in
How many zeros does the function y=x^2-8x-20 have
jenyasd209 [6]
The answer has 131;1 minutes
6 0
2 years ago
Jake bought 2 shirts and 1 pair of pants for $6. Mike bought 1 shirt and 1 pair of pants for $4. How much is each shirt and each
EastWind [94]

The cost of a shirt and a pair of pants is the same and is equal to $2. The correct option is B.

<h3>What is a solution to a system of equations?</h3>

For a solution to be the solution to a system, it must satisfy all the equations of that system, and as all points satisfying an equation are in their graphs, so solution to a system is the intersection of all its equation at a single point(as we need a common point, which is going to be the intersection of course)(this can be one or many, or sometimes none)

Let x represent the cost of a shirt, while y represent the cost of a pair of pants.

Given that Jake bought 2 shirts and 1 pair of pants for $6. Therefore, the equation that represents this situation is,

2x + y = 6 ..... equation 1

Also, Mike bought 1 shirt and 1 pair of pants for $4. Therefore, the equation that represents this situation is,

x + y = 4

Solve the equation for x,

x = 4 - y

Substitute the value of x in equation 1,

2x + y = 6

2(4 - y) + y = 6

8 - 2y + y = 6

-y = 6 - 8

-y = -2

y = 2

Substitute the value of y in equation 1,

2x + y = 6

2x + 2 = 6

2x + 2 = 6

2x = 6 - 2

2x = 4

x = 4/2

x = 2

Hence, the cost of a shirt and a pair of pants is the same and is equal to $2.

Learn more about finding the solution graphically here:

brainly.com/question/26254258

#SPJ1

6 0
1 year ago
What is the distance between (-5, 3) and (-5,-3)?
aev [14]
The answer is 6
hope this helped
5 0
2 years ago
What is the greatest value of 2,463.9051
RoseWind [281]

Hey There @Bre18016,

The answer is \boxed{2}

The greatest value of 2,463.9051 would be the thousands place (2) simply as it is the biggest number out of the other places.

For instance, if we had the number 300, 3 would be the greatest value.

Or let's say we had 10,000 the 1 would be the greatest value.

Furthermore, you could look at the first digit in the entire number to deter mine the greatest value.

6 0
2 years ago
In ΔEFG, the measure of ∠G=90°, the measure of ∠F=23°, and FG = 31 feet. Find the length of GE to the nearest tenth of a foot.
alexira [117]

Answer:

x=13.1587≈13.2 feet

6 0
3 years ago
Other questions:
  • What is the reciprocal of -1
    9·1 answer
  • Data from an independent research company found that the annual cost per worker for insurance​ (health, life,​ liability, etc.)
    11·1 answer
  • Converting with Decimals, Percent, and Fractions.<br><br> Please help me ASAP!
    6·1 answer
  • Find the balance in the account. $800 principal earning 7%, compounded annually, after 4 years
    6·1 answer
  • If a spring stretches 0.8 with 20 pounds attached how far will it stretch with 60 pounds attached?
    13·2 answers
  • What is 82 divided by 4 in whole numbers
    9·1 answer
  • PLeAsE hUrRy!
    12·1 answer
  • What is the value 7?
    13·1 answer
  • In a group of Year 9 students, the ratio of boys to girls is 9 : 7. There are 4 more boys than girls in this group. How many stu
    15·1 answer
  • Chad is making an orange smoothie. The recipe calls for three part frozen yogurt to five part orange juice. How many ounces of f
    6·1 answer
Add answer
Login
Not registered? Fast signup
Signup
Login Signup
Ask question!